leehuan's All-Levels-Of-Maths SOS thread (4 Viewers)

Status
Not open for further replies.

leehuan

Well-Known Member
Joined
May 31, 2014
Messages
5,805
Gender
Male
HSC
2015
Try using L'Hôpital's rule or a substitution.
Can't use the former cause haven't actually been taught it yet (or I would've). Will try a substitution later I guess, thanks.
 

Drongoski

Well-Known Member
Joined
Feb 22, 2009
Messages
4,251
Gender
Male
HSC
N/A


cancelling the common factors, as you were taught to do in your HSC - but were not told why you can do this. You would now know the reason, from learning the delta-epsilon definition of a limit.

To use L'Hopital's rule, you simply differentiate the numerator and the denominator wrt 'x', and substitute x=64 in the resultant expression; if still indefinite, you can reapply the rule until no longer indefinite, like the original expression would have given you a so-called indefinite 0/0.
 
Last edited:

leehuan

Well-Known Member
Joined
May 31, 2014
Messages
5,805
Gender
Male
HSC
2015
I know how to use L'Hopital's; I've used it before. I just haven't been taught it in my course yet to use it in my homework.

But I see. I was looking for something to cancel out so I guess the fractional powers threw me off.
 

leehuan

Well-Known Member
Joined
May 31, 2014
Messages
5,805
Gender
Male
HSC
2015
I have no idea what's happening, so I'm just gonna say "Geometric Series"
It has absolutely nothing to do with geometric series; this is all related to nothing more than limits.
 

InteGrand

Well-Known Member
Joined
Dec 11, 2014
Messages
6,109
Gender
Male
HSC
N/A
The answer to b) is a correct one.

The answer in c) is a correct one too. To see this, let eps > 0, N = 1/eps^2, and suppose x > N.

Then x > 1/eps^2, which implies sqrt(x) > 1/eps. This implies that sqrt(x) > (1/eps) – 1, and this is equivalent to 1/(1 + sqrt(x)) < eps, i.e. |1/(1+sqrt(x)) – 0| < eps. Hence if x > N, then |f(x) – L| < eps, so the N is a correct answer.
 

InteGrand

Well-Known Member
Joined
Dec 11, 2014
Messages
6,109
Gender
Male
HSC
N/A
An easier way to see that N works is that for any 0 < eps < 1, we have 1/eps > (1/eps – 1) > 0, so (1/eps^2) > (1/eps – 1)^2, that is, N > M. Since we know that M works (by 'works', I mean that x being greater than it will imply that |f(x) – L| < eps), taking any larger value (e.g. N) will also work. This is because if x > N, then x > M (as N > M), implying that |f(x) – L| < eps, so N works too.
 

leehuan

Well-Known Member
Joined
May 31, 2014
Messages
5,805
Gender
Male
HSC
2015
Ohhh should've trusted my very first instinct then. Thanks
 

leehuan

Well-Known Member
Joined
May 31, 2014
Messages
5,805
Gender
Male
HSC
2015
Lost in my algebra today big time.



textbf - what I'm stuck on basically
 

leehuan

Well-Known Member
Joined
May 31, 2014
Messages
5,805
Gender
Male
HSC
2015
Nothing to do with instinct. If you had said 'Yes', you'd still have to justify your answer.



Not sure if you understood what I meant.

My first instinct processed almost the ENTIRE method. But for some reason I had a contradiction at one point in my mind which halted me. And for some reason I went with that.
 

Drongoski

Well-Known Member
Joined
Feb 22, 2009
Messages
4,251
Gender
Male
HSC
N/A
Not sure if you understood what I meant.

My first instinct processed almost the ENTIRE method. But for some reason I had a contradiction at one point in my mind which halted me. And for some reason I went with that.
Ok
 

Green Yoda

Hi Φ
Joined
Mar 28, 2015
Messages
2,859
Gender
Male
HSC
2017
Question:
Determine the centre and radius of the circle.
x^2+y^2-18x+20y+60=0
 
Status
Not open for further replies.

Users Who Are Viewing This Thread (Users: 0, Guests: 4)

Top